Question about Vector Fields and Line Integrals

Click For Summary
The discussion revolves around evaluating line integrals in vector fields. For the first part, it is established that if the vector field F is normal to the curve C at every point, the line integral I equals zero because the dot product of F and the tangent vector c'(t) is zero. In the second part, when F is a unit vector pointing in the negative direction along the curve, the integral can be expressed as the integral of the negative derivative of the position vector divided by its magnitude, leading to a result of -c. The participants clarify that the relevant vectors are the tangent vector and the vector field, not the position vector. The discussion concludes with an emphasis on understanding the relationship between the numerator and denominator in the integral expression.
Mohamed Abdul

Homework Statement


(a) Consider the line integral I = The integral of Fdr along the curve C

i) Suppose that the length of the path C is L. What is the value of I if the vector field F is normal to C at every point of C?
ii) What is the value of I if the vector field F is is a unit vector pointing in the negative direction along the curve at every point of C?

Homework Equations


Integral of Fdr along c is F(r(t)) * r'(t)

The Attempt at a Solution


I understand the process of computing a line integral, but am unsure of these two parts.

I know for i) that a vector times its normal vector is 0, but F isn't r, it is a vector field of r.

For ii) I'm not even sure as to how to proceed. I do not know what my F(r(t)) would be nor my r'(t)
 
Physics news on Phys.org
Mohamed Abdul said:
I know for i) that a vector times its normal vector is 0, but F isn't r, it is a vector field of r.
##\vec F## is not normal to ##\vec r##, it is normal to the curve ##C##.

Mohamed Abdul said:
For ii) I'm not even sure as to how to proceed. I do not know what my F(r(t)) would be nor my r'(t)
You do not need to know this. Write down an expression for the integral and work with what you have.
 
Orodruin said:
##\vec F## is not normal to ##\vec r##, it is normal to the curve ##C##.You do not need to know this. Write down an expression for the integral and work with what you have.
So then for the first problem I would parametrize c and so then I would have the integral of the vector field of that c(t) times the derivative of c. And since the vector field is normal to c at every point would that mean that F(c(t)) * c'(t) would be 0?

As for the second one, I got that it would be the integral of c(t)/The magnitude of c(t) times c'(t). Am I on the right track with that?
 
Mohamed Abdul said:
As for the second one, I got that it would be the integral of c(t)/The magnitude of c(t) times c'(t). Am I on the right track with that?
##\vec c(t)## (the position vector in your notation) is not the relevant vector according to the problem statement. The field is supposed to be anti-parallel to the tangent vector, not the position vector.
Mohamed Abdul said:
So then for the first problem I would parametrize c and so then I would have the integral of the vector field of that c(t) times the derivative of c. And since the vector field is normal to c at every point would that mean that F(c(t)) * c'(t) would be 0?
Yes.
 
Orodruin said:
##\vec c(t)## (the position vector in your notation) is not the relevant vector according to the problem statement. The field is supposed to be anti-parallel to the tangent vector, not the position vector.

Yes.
So then my vector field wouldn't be the unit vector of c but the negative derivative of that unit vector since unlike in the first part, F is pointing in a certain direction.

So I would get the integral of -c'(t)/the magnitude of c'(t) multiplied by c'(t). Looking at the solution, the answer was supposed to be -c, but I am unsure of how this integral resolves to that value.
 
Mohamed Abdul said:
integral of -c'(t)/the magnitude of c'(t) multiplied by c'(t)
In other words
$$
- \int_C \frac{\vec c' \cdot \vec c'}{|\vec c'|} dt.
$$
What can you say about the numerator here and its relation to the denominator?
 
Question: A clock's minute hand has length 4 and its hour hand has length 3. What is the distance between the tips at the moment when it is increasing most rapidly?(Putnam Exam Question) Answer: Making assumption that both the hands moves at constant angular velocities, the answer is ## \sqrt{7} .## But don't you think this assumption is somewhat doubtful and wrong?

Similar threads

  • · Replies 23 ·
Replies
23
Views
3K
Replies
6
Views
3K
  • · Replies 2 ·
Replies
2
Views
2K
  • · Replies 20 ·
Replies
20
Views
3K
  • · Replies 7 ·
Replies
7
Views
2K
  • · Replies 8 ·
Replies
8
Views
2K
Replies
12
Views
2K
  • · Replies 6 ·
Replies
6
Views
2K
  • · Replies 5 ·
Replies
5
Views
1K